Đến nội dung

Hình ảnh

BĐT -Tuyển tập các bài toán sưu tầm từ Mathlinks.ro

* * * * - 14 Bình chọn tuyển tập sưu tầm.

  • Please log in to reply
Chủ đề này có 74 trả lời

#1
dark templar

dark templar

    Kael-Invoker

  • Hiệp sỹ
  • 3788 Bài viết
Thân chào tất cả các bạn :)

Thay cho lời mở đầu,mời các bạn đọc qua topic sau.

Nào,chúng ta cùng vào vấn đề chính.

Bài toán 1: Với 3 số dương $a,b,c$;hãy chứng minh rằng :
$$\frac{a^2}{b}+\frac{b^2}{c}+\frac{c^2}{a} \ge a+b+c+\frac{4(a-b)^2}{a+b+c}$$
"Do you still... believe in me ?" Sarah Kerrigan asked Jim Raynor - Starcraft II:Heart Of The Swarm.

#2
henry0905

henry0905

    Trung úy

  • Thành viên
  • 892 Bài viết

Thân chào tất cả các bạn :)

Thay cho lời mở đầu,mời các bạn đọc qua topic sau.

Nào,chúng ta cùng vào vấn đề chính.

Bài toán 1: Với 3 số dương $a,b,c$;hãy chứng minh rằng :
$$\frac{a^2}{b}+\frac{b^2}{c}+\frac{c^2}{a} \ge a+b+c+\frac{4(a-b)^2}{a+b+c}$$ (1)

Phát đầu:
$(1)\Leftrightarrow \sum (\frac{a^{2}}{b}-2a+b)\geq \frac{4(a-b)^{2}}{a+b+c}$
VT= $\Leftrightarrow \Leftrightarrow \sum \frac{(a-b)^{2}}{b}\geq \frac{(a-b+c-b+a-c)^{2}}{a+b+c}=\frac{4(a-b)^{2}}{a+b+c}$
Ta có dpcm.

#3
dark templar

dark templar

    Kael-Invoker

  • Hiệp sỹ
  • 3788 Bài viết

Phát đầu:
$(1)\Leftrightarrow \sum (\frac{a^{2}}{b}-2a+b)\geq \frac{4(a-b)^{2}}{a+b+c}$
VT= $\Leftrightarrow \Leftrightarrow \sum \frac{(a-b)^{2}}{b}\geq \frac{(a-b+c-b+a-c)^{2}}{a+b+c}=\frac{4(a-b)^{2}}{a+b+c}$
Ta có dpcm.

Mong em đọc kỹ dòng này .

Và cũng mong các bạn đưa ra một lời giải hoàn chỉnh ,đừng chỉ nêu hướng giải.

Nhấn mạnh ở chữ hoàn chỉnh đấy nhé ;)
"Do you still... believe in me ?" Sarah Kerrigan asked Jim Raynor - Starcraft II:Heart Of The Swarm.

#4
WhjteShadow

WhjteShadow

    Thượng úy

  • Phó Quản lý Toán Ứng dụ
  • 1323 Bài viết

Mong em đọc kỹ dòng này .

Nhấn mạnh ở chữ hoàn chỉnh đấy nhé ;)

:") E thấy bạn làm được rùi mà anh :D Bài này quá quen thuộc...
Đề xuất 1 bài toán tương tự (Tất nhiên là hay hơn rùi :">)
Đề nghị 1.1.
Ch0 các số thực không âm $a,b,c$ thỏa mãn $ab+bc+ca>0$. Chứng minh rằng:
$$\frac{a}{b+c}+\frac{b}{c+a}+\frac{c}{a+b}\geq \frac{3}{2}+\frac{7(a-c)^2}{16(ab+bc+ca)}$$
Ngoài điểm rơi $a=b=c$ còn có $a=\frac{4}{3}b,c=0$ :luoi:
“There is no way home, home is the way.” - Thich Nhat Hanh

#5
Mai Xuan Son

Mai Xuan Son

    Vagrant

  • Thành viên
  • 274 Bài viết

:") E thấy bạn làm được rùi mà anh :D Bài này quá quen thuộc...
Đề xuất 1 bài toán tương tự (Tất nhiên là hay hơn rùi :">)
Đề nghị 1.1.
Ch0 các số thực không âm $a,b,c$ thỏa mãn $ab+bc+ca>0$. Chứng minh rằng:
$$\frac{a}{b+c}+\frac{b}{c+a}+\frac{c}{a+b}\geq \frac{3}{2}+\frac{7(a-c)^2}{16(ab+bc+ca)}$$
Ngoài điểm rơi $a=b=c$ còn có $a=\frac{4}{3}b,c=0$ :luoi:

1.2 =))

$a,b,c\geq 0$
Prove
$\sum_{cyc}\frac{a}{b+c}\geq \frac{3}{2}+\frac{7}{16}.\frac{max{[(a-b)^2;(b-c)^2;(c-a)^2]}}{ab+bc+ac}$

Solution
Giả sử $a\geq b\geq c$
$\rightarrow$
$\sum \frac{a}{b+c}\geq \frac{3}{2}+\frac{7}{16}.\frac{(a-c)^2}{ab+bc+ca}$
$\Leftrightarrow \sum _{cyc}\frac{a[a(b+c)+bc]}{b+c}\geq \frac{3}{2}(ab+bc+ac)+\frac{17}{6}(a-c)^2$
$\Leftrightarrow \sum a^2+abc.\sum \frac{1}{b+c}\geq \frac{3}{2}(ab+bc+ac)+\frac{7}{16}(a-c)^2$
Theo AM-HM
$\frac{1}{a+c}+\frac{1}{b+c}+\frac{1}{a+b}\geq \frac{9}{2.(a+b+c)}$
Cần chứng minh
$a^2+b^2+c^2+\frac{9abc}{2(a+b+c)}\geq \frac{3}{2}(ab+bc+ac)+\frac{7}{16}(a-c)^2$
Từ trên đặt $a=c+x$
$b=c+y$ $(x\geq y\geq 0)$
Bất đẳng thức tương đương với:
..............
$(11x^2-32xy+32y^2)c+(x+y)(3x-4y)^2\geq 0$ ($TRUE$) $\blacksquare$
Dấu "=" khi
$a=b=c$ or $a=4/3b$ $c=0$

p/s: Đã dự được điểm rơi :ukliam2:

Bài viết đã được chỉnh sửa nội dung bởi tops2liz: 17-03-2013 - 07:28

~~~like phát~~~

#6
dark templar

dark templar

    Kael-Invoker

  • Hiệp sỹ
  • 3788 Bài viết

Phát đầu:
$(1)\Leftrightarrow \sum (\frac{a^{2}}{b}-2a+b)\geq \frac{4(a-b)^{2}}{a+b+c}$
VT= $\Leftrightarrow \Leftrightarrow \sum \frac{(a-b)^{2}}{b}\geq \frac{(a-b+c-b+a-c)^{2}}{a+b+c}=\frac{4(a-b)^{2}}{a+b+c}$
Ta có dpcm.

Anh nhấn mạnh ở chữ hoàn chỉnh là có lý do nhé Đạt ;)

Đáng lẽ nếu áp dụng C-S thì phải ra thế này :
$$VT \ge \frac{(|a-b|+|b-c|+|c-a|)^2}{a+b+c}$$

Và khi này với không mất tính tổng quát ta giả sử $|a-b|=\min \{|a-b|;|b-c|;|c-a| \}$ thì ngay lập tức ta có :
$$|a-b|+|b-c|+|c-a| \ge 2|a-b|$$

Từ đây ,ta cũng có đpcm.Đẳng thức xảy ra khi $a=b=c$ hay $a=\frac{\sqrt{5}-1}{2}c;c=\frac{\sqrt{5}-1}{2}b$.

Đồng thời cũng cảm ơn Đạt và tops2liz đã đưa ra 2 BĐT đề nghị.
**********
Mình sẽ đưa ra 1 cách giải khác cho bài toán 1:

Lời giải bài toán 1:
Ta có 1 đẳng thức sau :
$$\frac{a^2}{b}+\frac{b^2}{c}+\frac{c^2}{a}-(a+b+c)=\frac{1}{a+b+c}\left[\frac{(ac-b^2)^2}{bc}+\frac{(bc-a^2)^2}{ab}+\frac{(ab-c^2)^2}{ac} \right]$$

Do đó ta chỉ cần chứng minh:
$$\frac{(ac-b^2)^2}{bc}+\frac{(bc-a^2)^2}{ab}+\frac{(ab-c^2)^2}{ac} \ge 4(a-b)^2$$

Nhưng $\frac{(ab-c^2)^2}{ac} \ge 0$ và:
$$\frac{(ac-b^2)^2}{bc}+\frac{(bc-a^2)^2}{ab} \ge \frac{(|ac-b^2|+|bc-a^2|)^2}{b(a+c)} \ge \frac{(a-b)^2(a+b+c)^2}{b(a+c)}$$

Bằng AM-GM,ta có :
$$b(a+c) \le \frac{(a+b+c)^2}{4} \implies \frac{(a-b)^2(a+b+c)^2}{b(a+c)} \ge 4(a-b)^2$$

Từ đây,ta có đpcm.
**********
Tiếp tục với đề mới nào :

Bài toán 2: Cho $a,b,c,d,e$ là các số thực không âm và có tổng bằng 5.Chứng minh rằng: (7)
$$abc+bcd+cde+dea+eab \le 5$$

Bài toán 3: Cho $a,b,c,d>0$ thỏa mãn tổng bằng 4.Chứng minh rằng : (37)
$$\frac{1}{11+a^2}+\frac{1}{11+b^2}+\frac{1}{11+c^2}+\frac{1}{11+d^2} \le \frac{1}{3}$$

:)
"Do you still... believe in me ?" Sarah Kerrigan asked Jim Raynor - Starcraft II:Heart Of The Swarm.

#7
Ispectorgadget

Ispectorgadget

    Nothing

  • Quản lý Toán Phổ thông
  • 2946 Bài viết

Bài toán 2: Cho $a,b,c,d,e$ là các số thực không âm và có tổng bằng 5.Chứng minh rằng: (7)
$$abc+bcd+cde+dea+eab \le 5$$

:)

Giả sử $e=\min \{a,b,c,d,e\}$

Áp dụng BĐT Cauchy ta có
$$abc+bcd+cde+dea+eab=e(a+c)(b+d)+bc(a+c-e)\le e\left(\frac{a+b+c+d}{2} \right )^2+\left(\frac{b+c+a+d-e}{3} \right )^2 =\frac{e(5-e)^2}{4}+\frac{(5-2e)^3}{27}$$

Ta cần chứng minh $$\frac{e(5-e)^2}{4}+\frac{(5-2e)^3}{27}\le 1$$

Bất đẳng thức này tương đương với $$(e-1)^2(e+8)\ge 0. \square$$

Bài viết đã được chỉnh sửa nội dung bởi Ispectorgadget: 17-03-2013 - 10:42

►|| The aim of life is self-development. To realize one's nature perfectly - that is what each of us is here for. ™ ♫


#8
Joker9999

Joker9999

    Thiếu úy

  • Thành viên
  • 659 Bài viết

Bài toán 3: Cho $a,b,c,d>0$ thỏa mãn tổng bằng 4.Chứng minh rằng : (37)
$$\frac{1}{11+a^2}+\frac{1}{11+b^2}+\frac{1}{11+c^2}+\frac{1}{11+d^2} \le \frac{1}{3}$$


Bài này có vẻ dễ, hình như bỏ đk >0 đj cũng được
Ta chứng minh: $\frac{1}{11+a^2}\leq \frac{1}{12}-\frac{1}{72}(a-1)$, điều này tương đương với: $\frac{(a-1)^2(60-12a)}{864(11+a^2)}\geq 0$ đúng do $a\leq 4$
Thiết lập các BDT tương tự có đpcm
Đẳng thức xảy ra khi và chỉ khi $a=b=c=d=1$

Bài viết đã được chỉnh sửa nội dung bởi Joker9999: 17-03-2013 - 17:13

<span style="font-family: trebuchet ms" ,="" helvetica,="" sans-serif'="">Nỗ lực chưa đủ để thành công.


.if i sad, i do Inequality to become happy. when i happy, i do Inequality to keep happy.

#9
dark templar

dark templar

    Kael-Invoker

  • Hiệp sỹ
  • 3788 Bài viết

Bài này có vẻ dễ, hình như bỏ đk >0 đj cũng được
Ta chứng minh: $\frac{1}{11+a^2}\leq \frac{1}{12}-\frac{1}{72}(a-1)$, điều này tương đương với: $\frac{(a-1)^2(60-12a)}{864(11+a^2)}\geq 0$ đúng do $a\leq 4$
Thiết lập các BDT tương tự có đpcm
Đẳng thức xảy ra khi và chỉ khi $a=b=c=d=1$

Cách giải này phải cần đến điều kiện dương thì mới suy ra được $a<4$ ;) Bài này có nhiều cách lắm,cứ tìm tòi thử đi nhé :)
"Do you still... believe in me ?" Sarah Kerrigan asked Jim Raynor - Starcraft II:Heart Of The Swarm.

#10
dark templar

dark templar

    Kael-Invoker

  • Hiệp sỹ
  • 3788 Bài viết


Bài toán 2: Cho $a,b,c,d,e$ là các số thực không âm và có tổng bằng 5.Chứng minh rằng: (7)
$$abc+bcd+cde+dea+eab \le 5$$

Bài toán 3: Cho $a,b,c,d>0$ thỏa mãn tổng bằng 4.Chứng minh rằng : (37)
$$\frac{1}{11+a^2}+\frac{1}{11+b^2}+\frac{1}{11+c^2}+\frac{1}{11+d^2} \le \frac{1}{3}$$

:)

Lời giải cho bài toán 2 hoàn toàn giống với Kiên đã trình bày :)

Các bạn vẫn có thể tiếp tục thảo luận các bài toán cũ ngay cả khi có đề mới. :)

Lời giải bài toán 3:

Lời giải 1: (của Apartim De)
Xét hàm số $f(x)=\frac{1}{11+x^2}$ có đạo hàm cấp 2 là $f'(x)=\frac{6\left(x^2-\frac{11}{3} \right)}{(11+x^2)^3}$.

Do đó nếu $x \in \left(-\sqrt{\frac{11}{3}};\sqrt{\frac{11}{3}} \right)$ thì $f''(x)<0$,suy ra $f(x)$ là hàm lõm trong đoạn $\left(-\sqrt{\frac{11}{3}};\sqrt{\frac{11}{3}} \right)$.

Không mất tính tổng quát,giả sử $a \le b \le c \le d$.

Nếu cả 4 số $a,b,c,d$ đều thuộc khoảng $\left(0;\sqrt{\frac{11}{3}} \right)$ thì BĐT cần chứng minh sẽ đúng theo BĐT Jensen.

Do đó,xét có ít nhất 2 trong số 4 biến $a,b,c,d$ lớn hơn $\sqrt{\frac{11}{3}}$,giả sử đó là $c$ và $d$.

Dễ thấy rằng hàm số $f(x)$ ở trên là hàm nghịch biến trên khoảng xác định là $(0;+\infty)$,nên :
$$f(a)+f(b)+f\left(c \right)+f(d)<f(a-1)+f(b-1)+f(c-3)+f(d-3)<4f\left(\frac{a+b+c+d-8}{4} \right)=4f(-1)=\frac{1}{3}$$.

Lời giải 2: (của Popa Alexandru)
Viết BĐT lại dưới dạng tương đương sau : $\sum_{cyc}(1-a)\frac{a+1}{11+a^2} \ge 0$.

Để ý rằng nếu $a \ge b \ge c \ge d$ thì $\frac{a+1}{11+a^2} \le \frac{b+1}{11+b^2} \le \frac{c+1}{11+c^2} \le \frac{d+1}{11+d^2}$.

Do đó BĐT trên chỉ là hệ quả của BĐT Chebyshev.

**********
Đề mới :

Bài toán 4: Ký hiệu  thông thường $R,r,p$ trong tam giác.Chứng minh rằng $p^2 \ge 2R^2+8Rr+3r^2$.

Bài toán 5: Cho $0<a<b$ và $n$ số thực $x_1;x_2;...x_{n}$ thuộc đoạn $[a;b]$.Chứng minh :
$$\left(x_1+x_2+...+x_{n} \right)\left(\frac{1}{x_1}+\frac{1}{x_2}+...+\frac{1}{x_{n}} \right) \le \frac{n^2(a+b)^2}{4ab}$$

 


"Do you still... believe in me ?" Sarah Kerrigan asked Jim Raynor - Starcraft II:Heart Of The Swarm.

#11
Ispectorgadget

Ispectorgadget

    Nothing

  • Quản lý Toán Phổ thông
  • 2946 Bài viết





Bài toán 5: Cho $0<a<b$ và $n$ số thực $x_1;x_2;...x_{n}$ thuộc đoạn $[a;b]$.Chứng minh :
$$\left(x_1+x_2+...+x_{n} \right)\left(\frac{1}{x_1}+\frac{1}{x_2}+...+\frac{1}{x_{n}} \right) \le \frac{n^2(a+b)^2}{4ab}$$

 

$$VT =\left(\frac{a_1}{c}+\frac{a_2}{c}+...+\frac{a_n}{c} \right )\left(\frac{c}{a_1}+\frac{c}{a_2}+...+\frac{c}{a_n} \right )\le \frac{1}{4}\left(\frac{a_1}{c}+\frac{c}{a_1}+\frac{a_2}{c}+\frac{c}{a_2}+...+\frac{a_n}{c}+\frac{c}{a_n} \right )^2$$


Đặt $f(t)=\frac{c}{t}+\frac{t}{c}$

Ta có $f(t)$ đạt giá trị lớn nhất trên $[a;b]$ tại $a$ hoặc $b$.

Ta sẽ chọn $c$ sao cho $f(a)=f(b),c=\sqrt{ab}$

Vậy $f(t)\le \sqrt{\frac{a}{b}}+\sqrt{\frac{b}{a}}$

Vậy  $$VT \le n^2 \left( \sqrt{\frac{a}{b}}+\sqrt{\frac{b}{a}}\right )^2 =VP$$

 

Spoiler


►|| The aim of life is self-development. To realize one's nature perfectly - that is what each of us is here for. ™ ♫


#12
dark templar

dark templar

    Kael-Invoker

  • Hiệp sỹ
  • 3788 Bài viết


Bài toán 4: Ký hiệu  thông thường $R,r,p$ trong tam giác.Chứng minh rằng $p^2 \ge 2R^2+8Rr+3r^2$.

Bài toán 5: Cho $0<a<b$ và $n$ số thực $x_1;x_2;...x_{n}$ thuộc đoạn $[a;b]$.Chứng minh :
$$\left(x_1+x_2+...+x_{n} \right)\left(\frac{1}{x_1}+\frac{1}{x_2}+...+\frac{1}{x_{n}} \right) \le \frac{n^2(a+b)^2}{4ab}$$

 

Cứ tưởng sẽ có nhiều người ủng hộ topic này lắm,vì thấy trên diễn đàn khá nhiều topic BĐT...

Spoiler

 

Lời giải bài toán 4 (của Virgil Nicula):

Ta sẽ sử dụng các công thức liên qua đến các góc tam giác sau :

  • $(1):a^2+b^2+c^2=2(p^2-r^2-4Rr)$.
  • $(2):4S=(b^2+c^2-a^2)\tan A$.
  • $(3):\sin 2A+\sin 2B+\sin 2C=\frac{2S}{R^2}$.
  • $(4):\cos A+\cos B+\cos C=1+\frac{r}{R}$.

 

Do đó :

 

\[\begin{eqnarray*}\sum {{a^2}}  &=& \sum {\left( {{b^2} + {c^2} - {a^2}} \right)}=  4S\sum {\frac{{\cos A}}{{\sin A}}} \\&=& 8S\sum {\frac{{{{\cos }^2}A}}{{{\rm{sin2}}A}}} \ge  8S\frac{{{{\left( {\sum {\cos A} } \right)}^2}}}{{\sum {{\mathop{\rm s}\nolimits} {\rm{in2}}A} }}\\&=& 8S.\frac{{{{\left( {1 + \frac{r}{R}} \right)}^2}}}{{\frac{{2S}}{{{R^2}}}}} = 4{\left( {R + r} \right)^2}\\\Rightarrow {a^2} + {b^2} + {c^2} &\ge& 4{\left( {R + r} \right)^2}\\\Rightarrow 2\left( {{p^2} - {r^2} - 4Rr} \right) &\ge& 4{\left( {R + r} \right)^2}\\\Rightarrow {p^2} &\ge& 2{R^2} + 8Rr + 3{r^2}\end{eqnarray*}\]
 
Lời giải bài toán 5 (của Popa Alexandru):
Đặt $P = \left( {{x_1} + {x_2} + ... + {x_n}} \right)\left( {\frac{1}{{{x_1}}} + \frac{1}{{{x_2}}} + ... + \frac{1}{{{x_n}}}} \right)$.Ta có :
\[\begin{array}{rcl}P &=& \left( {{x_1} + {x_2} + ... + {x_n}} \right)\left( {\frac{1}{{{x_1}}} + \frac{1}{{{x_2}}} + ... + \frac{1}{{{x_n}}}} \right)\\&=& \left( {\frac{{{x_1}}}{c} + ... + \frac{{{x_n}}}{c}} \right)\left( {\frac{c}{{{x_1}}} + ... + \frac{c}{{{x_n}}}} \right)\\&\le& \frac{1}{4}{\left( {\frac{c}{{{x_1}}} + \frac{{{x_1}}}{c} + ... + \frac{c}{{{x_n}}} + \frac{{{x_n}}}{c}} \right)^2}\end{array}\]
 
Hàm số $f(t)=\frac{c}{t}+\frac{t}{c}$ đạt cực đại trên đoạn $[a;b]$ tại $a$ hay $b$.
 
Ta sẽ chọn $c$ sao cho $f(a)=f(b);c=\sqrt{ab}$.Từ đó $f(t) \le \sqrt{\frac{a}{b}}+\sqrt{\frac{b}{a}}$.
 
Suy ra :
\[P \le \frac{{{n^2}}}{4}{\left( {\sqrt {\frac{a}{b}}  + \sqrt {\frac{b}{a}} } \right)^2} = \frac{{{n^2}}}{4}\frac{{\left( {a + {b^2}} \right)}}{{ab}}\]
 
**********
Đề mới:
 
Bài toán 6: Cho $a,b,c \ge 0$.Chứng minh rằng :(35)
\[\left( {{a^2} - bc} \right)\sqrt {b + c}  + \left( {{b^2} - ca} \right)\sqrt {c + a}  + \left( {{c^2} - ab} \right)\sqrt {a + b}  \ge 0\]
 
Bài toán 7: Cho $x,y,z$ là các số thực thỏa mãn $3x+2y+z=1$.Tìm GTLN của $A=\frac{1}{1+|x|}+\frac{1}{1+|y|}+\frac{1}{1+|z|}$. (26)

"Do you still... believe in me ?" Sarah Kerrigan asked Jim Raynor - Starcraft II:Heart Of The Swarm.

#13
dark templar

dark templar

    Kael-Invoker

  • Hiệp sỹ
  • 3788 Bài viết

Bài toán 6: Cho $a,b,c \ge 0$.Chứng minh rằng :(35)


\[\left( {{a^2} - bc} \right)\sqrt {b + c}  + \left( {{b^2} - ca} \right)\sqrt {c + a}  + \left( {{c^2} - ab} \right)\sqrt {a + b}  \ge 0\]
 
Bài toán 7: Cho $x,y,z$ là các số thực thỏa mãn $3x+2y+z=1$.Tìm GTLN của $A=\frac{1}{1+|x|}+\frac{1}{1+|y|}+\frac{1}{1+|z|}$. (26)

Spoiler

Lời giải bài toán 6: 

 

Lời giải 1: (của Popa Alexandru

Đặt $\frac{a+b}{2}=x^2;...$ thì BĐT trở thành :

 

\[\begin{array}{l}\sum\limits_{cyc} {xy\left( {{x^3} + {y^3}} \right)}  \ge \sum\limits_{cyc} {{x^2}{y^2}\left( {x + y} \right)} \\\Leftrightarrow \sum\limits_{cyc} {xy\left( {x + y} \right){{\left( {x - y} \right)}^2}}  \ge 0\end{array}\]
 
Lời giải 2: (của Popa Alexandru)
Đặt $A=\sqrt{b+c};...$.Viết lại BĐT dưới dạng :
\[A\left( {{a^2} - bc} \right) + B\left( {{b^2} - ca} \right) + C\left( {{c^2} - ab} \right) \ge 0\]
 
Ta có :
\[\begin{array}{rcl}2\sum\limits_{cyc} {A\left( {{a^2} - bc} \right)}  &=& \sum\limits_{cyc} {A\left[ {\left( {a - b} \right)\left( {a + c} \right) + \left( {a - c} \right)\left( {a + b} \right)} \right]} \\&=& \sum\limits_{cyc} {A\left( {a - b} \right)\left( {a + c} \right)}  + \sum\limits_{cyc} {B\left( {b - a} \right)\left( {b + c} \right)} \\&=& \sum\limits_{cyc} {\left( {a - b} \right)\left[ {A\left( {a + c} \right) - B\left( {b + c} \right)} \right]} \\&=& \sum\limits_{cyc} {\left( {a - b} \right)\frac{{{A^2}{{\left( {a + c} \right)}^2} - {B^2}{{\left( {b + c} \right)}^2}}}{{A\left( {a + c} \right) + B\left( {b + c} \right)}}} \\&=& \sum\limits_{cyc} {\frac{{{{\left( {a - b} \right)}^2}\left( {a + c} \right)\left( {b + c} \right)}}{{A\left( {a + c} \right) + B\left( {b + c} \right)}}}  \ge 0\end{array}\]
 
Lời giải bài toán 7 (của dgreenb801):
Chúng ta có thể giả sử cả 3 số $x,y,z$ đều dương vì nếu 1 trong 3 số âm,chúng ta có thể làm cho nó dương và cho phép chúng ta giảm giá trị của 2 biến còn lại,từ đó làm cho tổng biểu thức lớn hơn.
 
Đặt $3x=a;2y=b;z=c$ thì $a+b+c=1$ và chúng ta cần tìm GTLN của $A=\frac{3}{{a + 3}} + \frac{2}{{b + 2}} + \frac{1}{{c + 1}}$.
 
Để ý rằng:
\[\left( {\frac{3}{{a + c + 3}} + 1} \right) - \left( {\frac{3}{{a + 3}} + \frac{1}{{c + 1}}} \right) = \frac{{{a^2}c + a{c^2} + 6ac + 6c}}{{\left( {a + 3} \right)\left( {c + 1} \right)\left( {a + c + 3} \right)}} \ge 0\]
 
Với $a+c$ thay đổi thì tổng đạt GTLN khi $c=0$.Ta cũng có thể lập luận tương tự với $b=0$ thì biểu thức A cũng đạt GTLN.
 
Vậy A đạt GTLN khi $a=1;b=c=0$,với GTLN là $\frac{11}{4}$.
 
**********
Đề mới :
 
Bài toán 8: Cho $a,b,c>0$.Chứng minh : (29)
\[\frac{{{a^3}}}{{{a^2} + {b^2} + ab}} + \frac{{{b^3}}}{{{b^2} + {c^2} + bc}} + \frac{{{c^3}}}{{{c^2} + {a^2} + ca}} \ge \frac{{a + b + c}}{3}\]
 
Bài toán 9: Cho $a,b,c,d>0$.Chứng minh : (33)
\[\frac{{{a^2} + {b^2}}}{{ab + {b^2}}} + \frac{{{b^2} + {c^2}}}{{bc + {c^2}}} + \frac{{{c^2} + {d^2}}}{{cd + {d^2}}} \ge \sqrt[3]{{\frac{{54a}}{{a + d}}}}\]

"Do you still... believe in me ?" Sarah Kerrigan asked Jim Raynor - Starcraft II:Heart Of The Swarm.

#14
nguyenthehoan

nguyenthehoan

    Sĩ quan

  • Thành viên
  • 392 Bài viết

**********

Đề mới :
 
Bài toán 8: Cho $a,b,c>0$.Chứng minh : (29)
\[\frac{{{a^3}}}{{{a^2} + {b^2} + ab}} + \frac{{{b^3}}}{{{b^2} + {c^2} + bc}} + \frac{{{c^3}}}{{{c^2} + {a^2} + ca}} \ge \frac{{a + b + c}}{3}\]
 

Áp dụng trực tiếp bất đẳng thức Svac ta có 

 

$VT\geq \frac{(\sum a^{2})^{2}}{\sum a^{3}+\sum a(b^{2}+c^{2})}$

 

Nên ta phải cm 

 

$\frac{(\sum a^{2})^{2}}{\sum a^{3}+\sum a(b^{2}+c^{2})}\geq \frac{\sum a}{3}$

 

$2(\sum a^{2})^{2}\geq 3(\sum a(b^{3}+c^{3}))$

 

Đúng vì ta có bất đẳng thức quen thuộc sau

 

$(\sum a^{2})^{2}\geq 3(a^{3}b+b^{3}c+c^{3}a)$

 

và $(\sum a^{2})^{2}\geq 3(b^{3}a+c^{3}b+a^{3}c)$ (dpcm)



#15
Idie9xx

Idie9xx

    Sĩ quan

  • Thành viên
  • 319 Bài viết

Spoiler

**********
Đề mới :
 
Bài toán 8: Cho $a,b,c>0$.Chứng minh : (29)
\[\frac{{{a^3}}}{{{a^2} + {b^2} + ab}} + \frac{{{b^3}}}{{{b^2} + {c^2} + bc}} + \frac{{{c^3}}}{{{c^2} + {a^2} + ca}} \ge \frac{{a + b + c}}{3}\]
 

Topic vắng ghê :mellow:

Một cách giải khác cho bài này.

Ta có $A=\sum \frac{a^3}{a^2 + b^2 + ab} - \sum \frac{b^3}{a^2 + b^2 + ab} =\sum \frac{a^3-b^3}{a^2 + b^2 + ab} = \sum (a-b)=0$

Xét thấy $3a^2-3ab+3b^2 \geq a^2+ab+b^2 \Leftrightarrow 3(a+b)(a^2-ab+b^2) \geq (a+b)(a^2+ab+b^2) \Leftrightarrow \frac{a^3+b^3}{a^2 + b^2 + ab} \geq \frac{a+b}{3}$

$\Rightarrow B=\sum \frac{a^3}{a^2 + b^2 + ab} + \sum \frac{b^3}{a^2 + b^2 + ab} \geq 2 \cdot \frac{a+b+c}{3}$

Nên $\sum \frac{a^3}{a^2 + b^2 + ab}=\frac{A+B}{2} \geq \frac{a+b+c}{3}$ (dpcm) :D

 

@Dark templar: Đây cũng là thủ thuật giải bài này của mình.Nhìn chung bài này có khá nhiều cách,các bạn cứ tiếp tục tìm cách khác nhé. :)


Bài viết đã được chỉnh sửa nội dung bởi dark templar: 07-04-2013 - 20:32

$\large \circ \ast R_f\cdot Q_r\cdot 1080\ast \circ$

#16
dark templar

dark templar

    Kael-Invoker

  • Hiệp sỹ
  • 3788 Bài viết


Bài toán 8: Cho $a,b,c>0$.Chứng minh : (29)


\[\frac{{{a^3}}}{{{a^2} + {b^2} + ab}} + \frac{{{b^3}}}{{{b^2} + {c^2} + bc}} + \frac{{{c^3}}}{{{c^2} + {a^2} + ca}} \ge \frac{{a + b + c}}{3}\]
 
Bài toán 9: Cho $a,b,c,d>0$.Chứng minh : (33)
\[\frac{{{a^2} + {b^2}}}{{ab + {b^2}}} + \frac{{{b^2} + {c^2}}}{{bc + {c^2}}} + \frac{{{c^2} + {d^2}}}{{cd + {d^2}}} \ge \sqrt[3]{{\frac{{54a}}{{a + d}}}}\]

 

Lời giải bài toán 8: 

 

Lời giải 1 (của Popa Alexandru):

Ta có:

\[\sum\limits_{cyc} {\frac{{{a^3}}}{{{a^2} + ab + {b^2}}}}  = \sum\limits_{cyc} {\frac{{{b^3}}}{{{a^2} + ab + {b^2}}}}  = \frac{1}{2}\sum\limits_{cyc} {\frac{{{a^3} + {b^3}}}{{{a^2} + ab + {b^2}}}} \]

 

Do đó ta chỉ cần chứng minh :

\[\frac{1}{2}\sum\limits_{cyc} {\frac{{{a^3} + {b^3}}}{{{a^2} + ab + {b^2}}}}  \ge \frac{{a + b + c}}{3}\]

 

Ta có:

 

\[\begin{array}{rcl}\frac{1}{2}\sum\limits_{cyc} {\frac{{{a^3} + {b^3}}}{{{a^2} + ab + {b^2}}}}  - \frac{{a + b + c}}{3} &=& \frac{1}{2}\sum\limits_{cyc} {\left( {\frac{{{a^3} + {b^3}}}{{{a^2} + ab + {b^2}}} - \frac{{a + b}}{3}} \right)} \\&=& \frac{1}{2}\sum\limits_{cyc} {\frac{{3{a^3} + 3{b^3} - {a^3} - {a^2}b - a{b^2} - {b^3} - {a^2}b - a{b^2}}}{{3\left( {{a^2} + ab + {b^2}} \right)}}} \\&=& \frac{1}{2}\sum\limits_{cyc} {\frac{{2{{\left( {a - b} \right)}^2}\left( {a + b} \right)}}{{3\left( {{a^2} + ab + {b^2}} \right)}}}  \ge 0\end{array}\]
 
Lời giải 2 (của Mohamed El-Alami):
Ta có:
\[\sum\limits_{cyc} {\frac{{{a^3}}}{{{a^2} + ab + {b^2}}}}  = \sum\limits_{cyc} {\frac{{{b^3}}}{{{a^2} + ab + {b^2}}}}  = \frac{1}{2}\sum\limits_{cyc} {\frac{{{a^3} + {b^3}}}{{{a^2} + ab + {b^2}}}} \]
 

Do $2(a^2+ab+b^2) \le 3(a^2+b^2)$ nên:

\[VT = \sum\limits_{cyc} {\frac{{{a^3} + {b^3}}}{{2\left( {{a^2} + ab + {b^2}} \right)}}}  \ge \sum\limits_{cyc} {\frac{{{a^3} + {b^3}}}{{3\left( {{a^2} + {b^2}} \right)}}}  \ge \frac{{a + b + c}}{3} = VP\]

 

 

Lời giải bài toán 9 (của Aravind Srinivas):

Việt lại VP dưới dạng sau $\frac{{{{\left( {\frac{a}{b}} \right)}^2} + 1}}{{\frac{a}{b} + 1}} + \frac{{{{\left( {\frac{b}{c}} \right)}^2} + 1}}{{\frac{b}{c} + 1}} + \frac{{{{\left( {\frac{c}{d}} \right)}^2} + 1}}{{\frac{c}{d} + 1}}$.

 

Dễ thấy hàm số $f(t)=\frac{t^2+1}{t+1}$ là hàm lồi với $t>0$ nên theo BĐT Jensen thì ta có:

\[VP \ge \frac{{{{\left( {\frac{{\frac{a}{b} + \frac{b}{c} + \frac{c}{a}}}{3}} \right)}^2} + 1}}{{\frac{a}{b} + \frac{b}{c} + \frac{c}{d} + 3}}\]

 

Đặt $K=\frac{a}{b}+\frac{b}{c}+\frac{c}{a}$ thì:

\[\frac{{{{\left( {\frac{K}{3}} \right)}^2} + 1}}{{K + 3}} = \frac{{K + \frac{9}{K}}}{{1 + \frac{3}{K}}} \ge \frac{6}{{1 + \frac{3}{K}}}\]

 

Mặt khác theo AM-GM $K \ge 3{\left( {\frac{d}{a}} \right)^{\frac{1}{3}}}$.

 

Do đó: 

\[\frac{6}{{1 + \frac{3}{K}}} \ge \frac{{6{a^{\frac{1}{3}}}}}{{{a^{\frac{1}{3}}} + {d^{\frac{1}{3}}}}} \ge \frac{{3{a^{\frac{1}{3}}}}}{{{{\left( {\frac{{a + d}}{2}} \right)}^{\frac{1}{3}}}}} = {\left( {\frac{{54a}}{{a + d}}} \right)^{\frac{1}{3}}}\]

 

**********

Đề mới:

 

Bài toán 10: Cho $a,b,c \ge 0$ có tổng bằng 1.Chứng minh: (32)

\[\frac{a}{{\sqrt {{b^2} + 3c} }} + \frac{b}{{\sqrt {{c^2} + 3a} }} + \frac{c}{{\sqrt {{a^2} + 3b} }} \ge \frac{1}{{\sqrt {1 + 3abc} }}\]

 

Bài toán 11: Cho $a,b,c>0$ thỏa mãn $a^3+b^3+3c=5$.Chứng minh: (41)

\[\sqrt {\frac{{a + b}}{{2c}}}  + \sqrt {\frac{{b + c}}{{2a}}}  + \sqrt {\frac{{c + a}}{{2b}}}  \le \frac{1}{a} + \frac{1}{b} + \frac{1}{c}\]

 

 

 

 

 


Bài viết đã được chỉnh sửa nội dung bởi dark templar: 13-04-2013 - 20:36

"Do you still... believe in me ?" Sarah Kerrigan asked Jim Raynor - Starcraft II:Heart Of The Swarm.

#17
Sagittarius912

Sagittarius912

    Trung úy

  • Thành viên
  • 776 Bài viết


 


 
Bài toán 8: Cho $a,b,c>0$.Chứng minh : (29)
\[\frac{{{a^3}}}{{{a^2} + {b^2} + ab}} + \frac{{{b^3}}}{{{b^2} + {c^2} + bc}} + \frac{{{c^3}}}{{{c^2} + {a^2} + ca}} \ge \frac{{a + b + c}}{3}\]
 
 

Bài này quen quen :))

Theo bất đẳng thức AM-GM,ta có

 

$\frac{a^3}{a^2+ab+b^2}=a-\frac{ab(a+b)}{a^2+ab+b^2}\ge a-\frac{a+b}{3}$

 

$\Rightarrow \sum \frac{a^3}{a^2+ab+b^2}\ge \sum (a-\frac{a+b}{2})=\frac{a+b+c}{3}$

 

Đây chính là đcpcm. Dấu đẳng thức xảy ra khi $a=b=c=$


Bài viết đã được chỉnh sửa nội dung bởi Sagittarius912: 14-04-2013 - 20:31


#18
WhjteShadow

WhjteShadow

    Thượng úy

  • Phó Quản lý Toán Ứng dụ
  • 1323 Bài viết

 

Bài toán 10: Cho $a,b,c \ge 0$ có tổng bằng 1.Chứng minh: (32)

\[\frac{a}{{\sqrt {{b^2} + 3c} }} + \frac{b}{{\sqrt {{c^2} + 3a} }} + \frac{c}{{\sqrt {{a^2} + 3b} }} \ge \frac{1}{{\sqrt {1 + 3abc} }}\]

 

Bài toán 11: Cho $a,b,c>0$ thỏa mãn $a^3+b^3+3c=5$.Chứng minh: (41)

\[\sqrt {\frac{{a + b}}{{2c}}}  + \sqrt {\frac{{b + c}}{{2a}}}  + \sqrt {\frac{{c + a}}{{2b}}}  \le \frac{1}{a} + \frac{1}{b} + \frac{1}{c}\]

 

Lâu nay để topic phủ bụi r` :D Quay lại thui :>

Bài toán 10:

Áp dụng bất đẳng thức Holder ta có :

$$\left(\sum_{cyc} \frac{a}{{\sqrt {{b^2} + 3c} }}\right)^2.\left[\sum_{cyc} a(b^2+3c)\right]\geq (a+b+c)^3=1$$

Vậy nên ta chỉ cần chứng minh :

$$\sum_{cyc} a(b^2+3c)\leq 1+3abc$$

Hay là :

$$ab^2+bc^2+ca^2+3(ab+bc+ca)\leq 1+3abc$$

Bất đẳng thức này hiển nhiên đúng do 3 bất đẳng thức quen thuộc :

$$ab^2+bc^2+ca^2\leq \frac{4}{27}-abc\,\,\,(\text{Bổ đề thân quen})$$

$$\frac{4(ab+bc+ca)-4}{9}\leq 4abc\,\,\,(\text{Schur})$$

$$\frac{23}{9}(ab+bc+ca)\leq \frac{23}{9}\,\,\,(\text{AM-GM})$$

Đẳng thức xảy ra tại $a=b=c=\frac{1}{3}$ $\blacksquare$

Bài toán 11:

Đầu tiên chúng ta đi khai thác cái giả thiết khá "lừa tình " :D

$$9=a^3+b^3+4+3c=(a^3+1+1)+(b^3+1+1)+3c\geq 3(a+b+c)$$

$$\Leftrightarrow a+b+c\leq 3$$

Công việc còn lại là phá căn bằng $AM-GM$ quá đơn giản :P

$$\sqrt {\frac{{a + b}}{{2c}}} + \sqrt {\frac{{b + c}}{{2a}}} + \sqrt {\frac{{c + a}}{{2b}}}\leq \frac{1}{2}.\sum \left(\frac{a+b}{2}+\frac{1}{c}\right)$$

$$\leq \frac{1}{2}.\left(3+\frac{1}{a}+\frac{1}{b}+\frac{1}{c}\right)\leq \frac{1}{a}+\frac{1}{b}+\frac{1}{c}$$

( Do $\frac{1}{a}+\frac{1}{b}+\frac{1}{c}\geq \frac{9}{a+b+c}\geq 3$ )

Kết thúc chứng minh. Dấu bằng xảy ra khi và chỉ khi $a=b=c=1$ $\square$


Bài viết đã được chỉnh sửa nội dung bởi WhjteShadow: 15-04-2013 - 12:01

“There is no way home, home is the way.” - Thich Nhat Hanh

#19
Poseidont

Poseidont

    Dark Knight

  • Thành viên
  • 322 Bài viết

Bài 10: Hoàn toàn có thể làm theo cách THCS, chậm hơn chú rồi

$VT=\sum_{cyc}\frac{a^2}{\sqrt{a}.\sqrt{ab^2+3ca}}\geq ^{Cauchy-Schwarz}\frac{(\sum a)^2}{\sum \sqrt{a}.\sqrt{ab^2+3ca}}\geq \frac{1}{\sqrt{\sum_{cyc} ab^2}+3\sum_{cyc}ab}$

Mà ta có

$\inline 1+3abc=(\sum _{cyc}a)^3+3abc=\sum_{cyc} a^3+3\prod_{cyc}(a+b)+3abc\geq \sum _{cyc}ab^2+\sum 3\sum _{cyc}a.\sum _{cyc}ab\geq \sum _{cyc}ab^2+\sum _{cyc}3ab$

Suy ra điều chứng minh


Nguyễn Đức Nghĩa tự hào là thành viên VMF


#20
dark templar

dark templar

    Kael-Invoker

  • Hiệp sỹ
  • 3788 Bài viết

2 lời giải của Đạt rất tuyệt,hoàn toàn giống với lời giải gốc. :)

 

Đề mới:

 

Bài toán 12: Cho $a,b,c>0$.Chứng minh : 

\[\frac{a}{b} + \frac{b}{c} + \frac{c}{a} \ge \frac{{c + a}}{{c + b}} + \frac{{a + b}}{{a + c}} + \frac{{b + c}}{{b + a}}\]

 

Bài toán 13: Cho $a,b,c,d>0$ thỏa mãn $a \le b \le c \le d$ và $abcd=1$.Chứng minh rằng:

\[\left( {a + 1} \right)\left( {d + 1} \right) \ge 3 + \frac{3}{{4{d^3}}}\]


Bài viết đã được chỉnh sửa nội dung bởi dark templar: 20-04-2013 - 20:59

"Do you still... believe in me ?" Sarah Kerrigan asked Jim Raynor - Starcraft II:Heart Of The Swarm.





Được gắn nhãn với một hoặc nhiều trong số những từ khóa sau: tuyển tập, sưu tầm.

1 người đang xem chủ đề

0 thành viên, 1 khách, 0 thành viên ẩn danh